LSAT and Law School Admissions Forum

Get expert LSAT preparation and law school admissions advice from PowerScore Test Preparation.

 Administrator
PowerScore Staff
  • PowerScore Staff
  • Posts: 8916
  • Joined: Feb 02, 2011
|
#25638
Complete Question Explanation
(The complete setup for this game can be found here: lsat/viewtopic.php?t=7800)

The correct answer choice is (A)

If G is visited first, and T – second, we need to ensure that G is visited at least one more time,
immediately before J (fourth rule). The first rule prohibits J from being visited fourth, while the
fourth rule also prohibits it from being visited third:
june07_game_3_#15_diagram_1.png
There appear to be only two possible weeks to visit J – 5 or 6. Recall, however, that G must be
always visited immediately before J, and also that it must be visited in some time between the two
visits to M. So, if the GJ Block were placed in weeks 5-6, then the two visits to M would be
adjacent to each other, in violation of both the third and fifth rules. Therefore, the GJ Block must
occupy the fourth and the fifth positions, flanked by the two M’s:
june07_game_3_#15_diagram_2.png
With only one possible solution, identifying answer choice (A) the correct answer to this Must Be
True question should be a breeze. The remaining four answer choices must be false.
You do not have the required permissions to view the files attached to this post.
 Haleyeastham
  • Posts: 33
  • Joined: Aug 03, 2015
|
#20172
This game refers to the 7 week long voyages. 15 is giving me some trouble. Can you please help explain? Thanks
 jeff.wren
PowerScore Staff
  • PowerScore Staff
  • Posts: 26
  • Joined: Jul 04, 2015
|
#20173
Hi Haley,

Game 3 of the June 2007 LSAT is a basic linear: unbalanced underfunded game where some of the variables will go more than once in order to fill in the 7 spaces. Since some of the variables go more than once, the game involves numerical distributions.

Question 15 is a local must be true question. Create a mini diagram with G in 1 and T in 2 (from the question) and T in 7 (from rule 2).

Like question 13, we need to consider the M>G>M rule and the GJ block.

The only way that these variables fit into the remaining four spaces is:

M in 3, GJ in 4 and 5, and M in 6.

Answer choice (A) correctly states that M is in 3.

Hope this helps,
Jeff

Get the most out of your LSAT Prep Plus subscription.

Analyze and track your performance with our Testing and Analytics Package.